Difference between revisions of "2024 AMC 12A Problems/Problem 18"
(→Solution 2) |
(→Solution 2) |
||
Line 32: | Line 32: | ||
==Solution 2== | ==Solution 2== | ||
+ | [[Image:2024_amc12A_p18.png|thumb|center|600px|]] | ||
+ | AC intersects BD at O, | ||
− | + | we want to find <math>\angle AOB </math> | |
− | since <math> | + | |
+ | since <math>tan(75^\circ) = 2+ \sqrt{3} = AD/AB </math>, <math>\angle CBD = \angle BCA = 15^\circ </math> | ||
<cmath>\angle AOB = \angle CBD + \angle BCA =30^\circ , </cmath> | <cmath>\angle AOB = \angle CBD + \angle BCA =30^\circ , </cmath> | ||
− | so each time rotate BD to AC for <math>30^\circ </math> | + | so each time we rotate BD to AC for <math>30^\circ </math>, |
− | and we need n = <math> 180^\circ / 30^\circ = 6 </math> | + | and we need to rotate n = <math> 180^\circ / 30^\circ = 6 </math> times to overlap with B (from one of A,B,C,D) |
( should not be n = <math>360^\circ / 30^\circ = 12) </math> | ( should not be n = <math>360^\circ / 30^\circ = 12) </math> | ||
<cmath>answer {(A) 6} </cmath> | <cmath>answer {(A) 6} </cmath> | ||
+ | |||
+ | note: if you don't remember <math>tan(75^\circ)</math> | ||
+ | <cmath> tan(75^\circ) = \frac{tan(45^\circ) + tan(30^\circ)}{ 1 - tan(45^\circ)*tan(30^\circ)} </cmath> | ||
+ | <cmath> = \frac{1 + \frac{1}{\sqrt{3}}}{1 - 1* \frac{1}{\sqrt{3}}} </cmath> | ||
+ | <cmath> = \frac{(\sqrt{3}+1)^2 }{ (\sqrt{3})^2-1} = 2+ \sqrt{3} </cmath> | ||
~[https://artofproblemsolving.com/wiki/index.php/User:Cyantist luckuso] | ~[https://artofproblemsolving.com/wiki/index.php/User:Cyantist luckuso] |
Revision as of 14:23, 9 November 2024
Contents
Problem
On top of a rectangular card with sides of length and , an identical card is placed so that two of their diagonals line up, as shown (, in this case).
Continue the process, adding a third card to the second, and so on, lining up successive diagonals after rotating clockwise. In total, how many cards must be used until a vertex of a new card lands exactly on the vertex labeled in the figure?
Solution 1
Let the midpoint of be .
We see that no matter how many moves we do, stays where it is.
Now we can find the angle of rotation () per move with the following steps:
Since Vertex is the closest one and
Vertex C will land on Vertex B when cards are placed.
(someone insert diagram maybe)
~lptoggled, minor Latex edits by eevee9406
Solution 2
AC intersects BD at O,
we want to find
since ,
so each time we rotate BD to AC for , and we need to rotate n = times to overlap with B (from one of A,B,C,D) ( should not be n =
note: if you don't remember
Solution 3(In case you have no time and that's what I did)
tan 15=sin15/cos15=1/(2+sqrt3) and it eliminates all options except 6 and 12. After one rotation it has turned 30degrees, so to satisfy the problem, divide 180 by 30 and you get 6
See also
2024 AMC 12A (Problems • Answer Key • Resources) | |
Preceded by Problem 17 |
Followed by Problem 19 |
1 • 2 • 3 • 4 • 5 • 6 • 7 • 8 • 9 • 10 • 11 • 12 • 13 • 14 • 15 • 16 • 17 • 18 • 19 • 20 • 21 • 22 • 23 • 24 • 25 | |
All AMC 12 Problems and Solutions |
The problems on this page are copyrighted by the Mathematical Association of America's American Mathematics Competitions.